Find linear transformation using diagonal matrix

Click For Summary
The discussion revolves around finding a linear transformation using a diagonal matrix for the transformation defined as L(x,y,z)=(y-z,x+z,-x+y). The matrix representation of L is provided, along with its diagonal form, which reveals the eigenvalues. It is concluded that L is not an isomorphism due to repeated eigenvalues. The user seeks clarification on how to compute L(1,0,0) using the diagonal matrix, indicating confusion about the relationship between eigenvalues and eigenvectors in this context. Assistance is requested to complete the final part of the homework, specifically regarding the linear combination of eigenvectors.
csc2iffy
Messages
74
Reaction score
0

Homework Statement


Define L:R3-->R3 by L(x,y,z)=(y-z,x+z,-x+y).
A. Show that L is self-adjoint using the standard orthonormal basis B of R3.
B. Diagonalize L and find and orthogonal basis B of R3 of eigenvectors of L and the diagonal matrix.
C. Considering only the eigenvalues of L, determine if L is an isomorphism.
D. Find L(1,0,0) using the diagonal matrix of L.


Homework Equations


L(x,y,z)=(y-z,x+z,-x+y)

Matrix of L with respect to orthonormal basis:
0 1 -1
1 0 1
-1 1 0

Diagonal matrix of L:
1 0 0
0 1 0
0 0 -2

The Attempt at a Solution


I already answered A and B. For C, I said L is not an isomorphism because of repeated eigenvalues. For D, I am not sure how to find the linear transformation using only the diagonal matrix, but I know the answer is (0,1,-1).
 
Physics news on Phys.org
A linear transformation is an isomorphism as long as it is invertible. What does that tell you about its eigenvalues? I'm not sure how to interpret "using the diagonal matrix". You cannot do this using only the diagonal matrix (the eigenvalues), you have to use the eigenvectors as well. This matrix is diagonalizable because there exist a basis for the space consisting of eigenvectors. Write (1, 0, 0) as a linear combination of eigenvectors.
 
thanks for your response! I'm pretty much done with the question besides part D.)
It says to find L(1,0,0) using the matrix in B.)
I'm not really sure how to do this.. I've been looking through my textbook for hours. Any extra help would be very much appreciated!

I tried writing it as a linear combination
|1 -1 1| |a| |1|
|1 0 -1| |b| = |0|
|0 1 1| |c| |0|
and I get
|a| | .333 |
|b| = |-.333 |
|c| | .333 |

and idk I'm getting lost... someone please help
 
Last edited:
Question: A clock's minute hand has length 4 and its hour hand has length 3. What is the distance between the tips at the moment when it is increasing most rapidly?(Putnam Exam Question) Answer: Making assumption that both the hands moves at constant angular velocities, the answer is ## \sqrt{7} .## But don't you think this assumption is somewhat doubtful and wrong?

Similar threads

  • · Replies 8 ·
Replies
8
Views
2K
Replies
9
Views
2K
  • · Replies 2 ·
Replies
2
Views
1K
Replies
4
Views
2K
  • · Replies 8 ·
Replies
8
Views
1K
  • · Replies 3 ·
Replies
3
Views
2K
Replies
5
Views
2K
  • · Replies 16 ·
Replies
16
Views
2K
  • · Replies 14 ·
Replies
14
Views
3K
  • · Replies 2 ·
Replies
2
Views
1K